Difference between revisions of "1996 AIME Problems/Problem 13"

m (Solution)
(Solution)
Line 21: Line 21:
  
 
Subtracting the two equations yields <math>DE\sqrt{57} + \frac{57}{4} = \frac{105}{4} \Longrightarrow DE = \frac{12}{\sqrt{57}}</math>. Then <math>\frac mn = \frac{1}{2} + \frac{DE}{2AE} = \frac{1}{2} + \frac{\frac{12}{\sqrt{57}}}{2 \cdot \frac{\sqrt{57}}{2}} = \frac{27}{38}</math>, and <math>m+n = \boxed{065}</math>.
 
Subtracting the two equations yields <math>DE\sqrt{57} + \frac{57}{4} = \frac{105}{4} \Longrightarrow DE = \frac{12}{\sqrt{57}}</math>. Then <math>\frac mn = \frac{1}{2} + \frac{DE}{2AE} = \frac{1}{2} + \frac{\frac{12}{\sqrt{57}}}{2 \cdot \frac{\sqrt{57}}{2}} = \frac{27}{38}</math>, and <math>m+n = \boxed{065}</math>.
 +
 +
 +
== Solution 2==
 +
Because the problem asks for a ratio, we can divide each side length by <math>\sqrt{3}</math> to make things simpler. We now have a triangle with sides <math>\sqrt{10}</math>, <math>\sqrt{5}</math>, and <math>\sqrt{2}</math>.
 +
 +
We use the same graph as above.
 +
 +
Draw perpendicular from <math>C</math> to <math>AE</math>. Denote this point as <math>F</math>. We know that <math>DE = EF = x</math> and <math>BD = CF = z</math> and also let <math>AE = y</math>.
 +
 +
Using Pythagorean theorem, we get three equations,
 +
 +
<center><math>(y+x)^2 + z^2 = 10</math>
 +
 +
<math>(y-x)^2 + z^2 = 2</math>
 +
 +
<math>x^2 + z^2 = \frac{5}{4}</math></center>
 +
 +
Adding the first and second, we obtain <math>x^2 + y^2 + z^2 = 6</math>, and then subtracting the third from this we find that <math>y = \frac{\sqrt{19}}{2}</math>. (Note, we could have used [[Stewart's Theorem]] to achieve this result).
 +
 +
Subtracting the first and second, we see that <math>xy = 2</math>, and then we find that <math>x = \frac{4}{\sqrt{19}}</math>
 +
 +
Using base ratios, we then quickly find that the desired ratio is <math>\frac{27}{38}</math> so our answer is <math>\boxed{65}</math>
  
 
== See also ==
 
== See also ==

Revision as of 01:17, 25 February 2016

Problem

In triangle $ABC$, $AB=\sqrt{30}$, $AC=\sqrt{6}$, and $BC=\sqrt{15}$. There is a point $D$ for which $\overline{AD}$ bisects $\overline{BC}$, and $\angle ADB$ is a right angle. The ratio $\frac{[ADB]}{[ABC]}$ can be written in the form $\dfrac{m}{n}$, where $m$ and $n$ are relatively prime positive integers. Find $m+n$.

Solution

[asy] pointpen = black; pathpen = black + linewidth(0.7); pair B=(0,0), C=(15^.5, 0), A=IP(CR(B,30^.5),CR(C,6^.5)), E=(B+C)/2, D=foot(B,A,E); D(MP("A",A)--MP("B",B,SW)--MP("C",C)--A--MP("D",D)--B); D(MP("E",E));  MP("\sqrt{30}",(A+B)/2,NW); MP("\sqrt{6}",(A+C)/2,SE); MP("\frac{\sqrt{15}}2",(E+C)/2); D(rightanglemark(B,D,A)); [/asy]

Let $E$ be the midpoint of $\overline{BC}$. Since $BE = EC$, then $\triangle ABE$ and $\triangle AEC$ share the same height and have equal bases, and thus have the same area. Similarly, $\triangle BDE$ and $BAE$ share the same height, and have bases in the ratio $DE : AE$, so $\frac{[BDE]}{[BAE]} = \frac{DE}{AE}$ (see area ratios). Now,

$\dfrac{[ADB]}{[ABC]} = \frac{[ABE] + [BDE]}{2[ABE]} = \frac{1}{2} + \frac{DE}{2AE}.$

By Stewart's Theorem, $AE = \frac{\sqrt{2(AB^2 + AC^2) - BC^2}}2 = \frac{\sqrt {57}}{2}$, and by the Pythagorean Theorem on $\triangle ABD, \triangle EBD$,

\begin{align*} BD^2 + \left(DE + \frac {\sqrt{57}}2\right)^2 &= 30 \\ BD^2 + DE^2 &= \frac{15}{4} \\ \end{align*}

Subtracting the two equations yields $DE\sqrt{57} + \frac{57}{4} = \frac{105}{4} \Longrightarrow DE = \frac{12}{\sqrt{57}}$. Then $\frac mn = \frac{1}{2} + \frac{DE}{2AE} = \frac{1}{2} + \frac{\frac{12}{\sqrt{57}}}{2 \cdot \frac{\sqrt{57}}{2}} = \frac{27}{38}$, and $m+n = \boxed{065}$.


Solution 2

Because the problem asks for a ratio, we can divide each side length by $\sqrt{3}$ to make things simpler. We now have a triangle with sides $\sqrt{10}$, $\sqrt{5}$, and $\sqrt{2}$.

We use the same graph as above.

Draw perpendicular from $C$ to $AE$. Denote this point as $F$. We know that $DE = EF = x$ and $BD = CF = z$ and also let $AE = y$.

Using Pythagorean theorem, we get three equations,

$(y+x)^2 + z^2 = 10$

$(y-x)^2 + z^2 = 2$

$x^2 + z^2 = \frac{5}{4}$

Adding the first and second, we obtain $x^2 + y^2 + z^2 = 6$, and then subtracting the third from this we find that $y = \frac{\sqrt{19}}{2}$. (Note, we could have used Stewart's Theorem to achieve this result).

Subtracting the first and second, we see that $xy = 2$, and then we find that $x = \frac{4}{\sqrt{19}}$

Using base ratios, we then quickly find that the desired ratio is $\frac{27}{38}$ so our answer is $\boxed{65}$

See also

1996 AIME (ProblemsAnswer KeyResources)
Preceded by
Problem 12
Followed by
Problem 14
1 2 3 4 5 6 7 8 9 10 11 12 13 14 15
All AIME Problems and Solutions

The problems on this page are copyrighted by the Mathematical Association of America's American Mathematics Competitions. AMC logo.png